LSAT and Law School Admissions Forum

Get expert LSAT preparation and law school admissions advice from PowerScore Test Preparation.

 Administrator
PowerScore Staff
  • PowerScore Staff
  • Posts: 8917
  • Joined: Feb 02, 2011
|
#26380
Complete Question Explanation
(The complete setup for this game can be found here: lsat/viewtopic.php?t=7717)

The correct answer choice is (C)

The question stem establishes an LG block, and such a block can only occur under Template #2. The appearance of the block creates the following sequential relationship:
powerscore_M12_T3_J2010_LG_explanations_game_2_#11_diagram_1.png
J must also appear later than P. Given the positioning of the actors, it is apparent that L must be one of the first three actors to appear (L-G-P, P-L-G, or P-M-L), and thus answer choice (C) is correct.
You do not have the required permissions to view the files attached to this post.

Get the most out of your LSAT Prep Plus subscription.

Analyze and track your performance with our Testing and Analytics Package.